\( \def\CC{{\mathbb C}} \def\RR{{\mathbb R}} \def\NN{{\mathbb N}} \def\ZZ{{\mathbb Z}} \def\TT{{\mathbb T}} \def\CF{{\operatorname{CF}^\bullet}} \def\HF{{\operatorname{HF}^\bullet}} \def\SH{{\operatorname{SH}^\bullet}} \def\ot{{\leftarrow}} \def\st{\;:\;} \def\Fuk{{\operatorname{Fuk}}} \def\emprod{m} \def\cone{\operatorname{Cone}} \def\Flux{\operatorname{Flux}} \def\li{i} \def\ev{\operatorname{ev}} \def\id{\operatorname{id}} \def\grad{\operatorname{grad}} \def\ind{\operatorname{ind}} \def\weight{\operatorname{wt}} \def\Sym{\operatorname{Sym}} \def\HeF{\widehat{CHF}^\bullet} \def\HHeF{\widehat{HHF}^\bullet} \def\Spinc{\operatorname{Spin}^c} \def\min{\operatorname{min}} \def\div{\operatorname{div}} \def\SH{{\operatorname{SH}^\bullet}} \def\CF{{\operatorname{CF}^\bullet}} \def\Tw{{\operatorname{Tw}}} \def\Log{{\operatorname{Log}}} \def\TropB{{\operatorname{TropB}}} \def\wt{{\operatorname{wt}}} \def\Span{{\operatorname{span}}} \def\Crit{\operatorname{Crit}} \def\into{\hookrightarrow} \def\tensor{\otimes} \def\CP{\mathbb{CP}} \def\eps{\varepsilon} \) SympSnip:

We parameterize the Lagrangian \(L\) by with the map \begin{align*} \li: Q\into& T^*Q && q\mapsto&(q, \alpha(q)) \end{align*} We now wish to show that \(i^*\omega=0\). Let \((q_1, \ldots q_n)\) be local coordinates on \(Q\), so that \(\alpha(q)=\sum_{k=1}^n \alpha_k(q)dq_{k}\). Let \(\partial_{q_i}, \partial_{q_j}\) be two basis vectors for the tangent space of \(Q\). Let \(\{(\partial_{q_i}, 0)\}\cup\{(0, \partial p_i)\}\) be a basis for the tangent space of \(T^*Q\). \begin{align*} \li^*\omega(\partial_{q_i}, \partial_{q_j})=& \omega \left(\left(\partial_{q_i}, \sum_{k=1}^n (\partial_{q_i}\alpha_k )\cdot \partial_{p_k}\right), \left(\partial_{q_j}, \sum_{k=1}^n (\partial_{q_j} \alpha_k)\cdot \partial_{p_k}\right)\right)\\ =&\left( \sum_{l=1}^n dp_l \wedge dq_l\right) \left(\left(\partial_i, \sum_{k=1}^n (\partial_{q_i}\alpha_k )\cdot \partial_{p_k}\right), \left(\partial_{q_j}, \sum_{k=1}^n (\partial_{q_j} \alpha_k )\cdot\partial_{p_k}\right)\right)\\ \end{align*} Since \(dq_i\partial_{q_j}=\delta_{ij}\) and \(dp_i\partial_{p_j}=\delta_{ij}\) \begin{align*} =& \partial_{q_j} \alpha_i - \partial_{q_i} \alpha_j\\ =& d\alpha(\partial_{q_i}, \partial_{q_j}) \end{align*} This vanishes for all \(i, j\) if and only if \(\alpha\) is closed.